0 Daumen
842 Aufrufe

Aufgabe:

Hallo

Kann jemand die Frage antworten.

Wie kann ich mit der dyadischen Zerlegung beweisen, dass die Reihe \( \sum\limits_{n=1}^{\infty}{\frac{1}{n^2}} \) Konvergenz ist?

Bestimmen Sie damit eine obere Schranke für die Summe der Reihe.

Avatar von

1 Antwort

0 Daumen
 
Beste Antwort

Aloha :)

Wenn wir den Nenner eines postiven Bruches verkleiner, wird der Bruch größer:$$\sum\limits_{n=1}^N\frac{1}{n^2}=\frac{1}{1^2}+\sum\limits_{n=2}^N\frac{1}{n^2}<1+\sum\limits_{n=2}^N\frac{1}{(n-1)\cdot n}=1+\sum\limits_{n=2}^N\left(\frac{1}{n-1}-\frac{1}{n}\right)$$$$\phantom{\sum\limits_{n=1}^N\frac{1}{n^2}}=1+\sum\limits_{n=2}^N\frac{1}{n-1}-\sum\limits_{n=2}^N\frac{1}{n}=1+\sum\limits_{n=1}^{N-1}\frac{1}{n}-\sum\limits_{n=2}^N\frac{1}{n}$$$$\phantom{\sum\limits_{n=1}^N\frac{1}{n^2}}=1+\left(\frac11+\sum\limits_{n=2}^{N-1}\frac{1}{n}\right)-\left(\sum\limits_{n=2}^{N-1}\frac{1}{n}+\frac{1}{N}\right)=2-\frac1N<2$$Für \(N\to\infty\) finden wir die obere Grenze \(2\) und die Konvergenz der Summe.

Avatar von 148 k 🚀

Ein anderes Problem?

Stell deine Frage

Willkommen bei der Mathelounge! Stell deine Frage einfach und kostenlos

x
Made by a lovely community